Converting Cartesian Coordinates to Cylindrical Coordinates

Click For Summary
To convert Cartesian coordinates to cylindrical coordinates, the equation y = -5 can be expressed as r sin(φ) = -5. For the point (0, -5), the cylindrical coordinates can be derived by recognizing that r = 5 and φ = 3π/2, since it lies on the negative y-axis. The discussion highlights the relationship between Cartesian and cylindrical coordinates, emphasizing the need to solve for r and φ using the equations x = r cos(φ) and y = r sin(φ). By manipulating these equations, one can effectively convert between the two coordinate systems. Understanding these conversions is crucial for accurately representing points and lines in different geometrical contexts.
seang
Messages
184
Reaction score
0

Homework Statement



Say I have line, y = - 5 in cartesian coordinates. How do I express this in cylindrical coordinates?

Also, if I have a point (0,-5) in cartesian coordinates, how to I express this position vector in cylindrical coordinates?

Homework Equations



y = r sin (phi)

j = r(hat)cos (phi) - phi(hat) sin (phi)


The Attempt at a Solution



THe relations are right there but I've forgotten howto use them.
 
Physics news on Phys.org
do you remember how to turn say (x,y) into (r,\theta) in 2D? the idea there is similar...
 
Cylindrical coordinates? That's in 3d but you only mention 2 coordinates? Perhaps you mean polar coordinates- cylindrical coordinates but ignore the z component.

You mention y= r sin(\phi) (I would have used \theta). Do you also know that x= r cos(\phi)? Solve those two equations for r and \phi in terms of x and y. The first is easy! You can eliminate \phi by squaring both equations and adding. The second is about as easy: eliminate r by dividing one equation by the other.
 
Question: A clock's minute hand has length 4 and its hour hand has length 3. What is the distance between the tips at the moment when it is increasing most rapidly?(Putnam Exam Question) Answer: Making assumption that both the hands moves at constant angular velocities, the answer is ## \sqrt{7} .## But don't you think this assumption is somewhat doubtful and wrong?

Similar threads

  • · Replies 4 ·
Replies
4
Views
2K
  • · Replies 6 ·
Replies
6
Views
2K
Replies
2
Views
1K
  • · Replies 2 ·
Replies
2
Views
2K
  • · Replies 14 ·
Replies
14
Views
2K
  • · Replies 4 ·
Replies
4
Views
2K
  • · Replies 6 ·
Replies
6
Views
3K
  • · Replies 6 ·
Replies
6
Views
3K
  • · Replies 7 ·
Replies
7
Views
2K
Replies
6
Views
2K